geometry pls helpppppp

Geometry Pls Helpppppp

Answers

Answer 1

A graph of triangle TOW is shown below.

A graph of triangle T'O'W, the image of ΔTOW after a reflection in the y-axis is shown below.

The coordinates of ΔT'O'W are: T' (5, 6), O' (-1, 8), and W' (4, -2).

What is a reflection over the y-axis?

In Mathematics and Geometry, a reflection over or across the y-axis is represented by this transformation rule (x, y) → (-x, y). This ultimately implies that, a reflection over or across the y-axis would maintain the same y-coordinate while the sign of the x-coordinate changes from positive to negative or negative to positive.

Next, we would apply a reflection over or across the x-axis;

(x, y)                →      (-x, y)

T (-5, 6)       →      (-(-5), 6) = T' (5, 6)

O (1, 8)       →      (-(-1), 8) = O' (-1, 8)

W (-4, -2)       →      (-(-4), -2) = W' (4, -2)

Read more on reflection here: brainly.com/question/27912791

#SPJ1

Geometry Pls Helpppppp
Geometry Pls Helpppppp

Related Questions

Need help pls and thank u! ​

Answers

Answer:

[tex]\sqrt[]{72}[/tex]

6.3141414141414...

[tex]\sqrt[4]{64}[/tex]

8.121 121 112 111...

Step-by-step explanation:

irrational numbers are real numbers that cannot be expressed as a ratio of integers.

example of irrational number [tex]\sqrt[]{2}[/tex]

example of rational number 2, 3, -4, etc.

Vanilla rent increased by 5%. The increase was $82. What was the original amount of Pavati's rent?

Answers

Let's call the original amount of Pavati's rent "x". If the rent increased by 5%, the new rent is 1.05x, which is the original rent plus a 5% increase. We know that the increase was $82, so we can set up the equation:

1.05x - x = 82

Simplifying the left side, we get:

0.05x = 82

Dividing both sides by 0.05, we get:

x = 1640

Therefore, the original amount of Pavati's rent was $1640.

Answer all questions please

Answers

The value of f(1) is 3.

An estimate of the value of f(-1) is -0.2.

The values of x for which f(x) = 1 are: x = (0, 3).

The value of x such that f(x) = 0 is x = -0.5.

The domain of f is {-2, 4}.

The range of f is {-1, 3}.

The interval over which f is increasing is {-2, 1}.

What is a domain?

In Mathematics and Geometry, a domain is the set of all real numbers for which a particular function is defined.

Furthermore, the vertical extent of any graph of a function represents all range values and they are always read and written from smaller to larger numerical values, and from the bottom of the graph to the top.

By critically observing the graph of the function shown in the image attached above, we can reasonably and logically deduce the following domain and range:

Domain = {-2, 4}.

Range = {-1, 3}.

In conclusion, we can logically deduce that this function is increasing over the [-2, 1] and decreasing over the interval [1, 4].

Read more on domain here: brainly.com/question/17440903

#SPJ1

Alice finds shirts on sale for $18.99.She buys twelve how much money does she spend?

Answers

Answer:

Well, if Alice buys 12 shirts which each cost $18.99 the equation would be 18.99 * 12 which = 227.88

Alice spent $227.88 on 12 shirts

Step-by-step explanation:

Answer:

Well, if Alice buys 12 shirts which each cost $18.99 the equation would be 18.99 * 12 which = 227.88

Step-by-step explanation:

A towns government is looking into its residences opinion on rebuilding the boardwalk on the coast line.
two representatives from the town visit the existing boardwalk and randomly survey 50 people to see whether they support the new boardwalk, they find that 60% of those surveyed support the construction of the new boardwalk and conclude with 90% confidence the majority of residents support its construction, what aspects of the scenario brings the validity of this conclusion into doubt

Answers

The aspects of the scenario that bring the validity of the conclusion made by the representatives of the government would be:

Small sample sizeSampling bias

What reduced the validity of the sample ?

The sample size used by the representatives in the survey was only 50 individuals, which might not be sufficient to represent the views of the entire town's population accurately. A more extensive sample size would provide a more precise approximation of the public opinion.

Moreover, the conductance of the survey on the existing boardwalk presents the possibility of sampling bias since those who visit the boardwalk could hold different opinions towards the new construction than those that do not visit.

Find out more on validity at https://brainly.com/question/24158610

#SPJ1

Determine the value of y, if x is 3.
y = x² + 11

Answers

Answer:

20

Step-by-step explanation:

just a substitute 3 in x so 3x3=9

11+9=20

How many times greater is the
value of the 4 in 2,849 than the
value of the 4 in 3,824?

Answers

Answer:

The value of the 4 in 2,849 is 4 x 100 = 400, while the value of the 4 in 3,824 is 4 x 10 = 40. Therefore, the value of the 4 in 2,849 is 400/40 = 10 times greater than the value of the 4 in 3,824.

Step-by-step explanation:

Forty people were asked their favorite kind of pizza. Thirty percent of the people surveyed chose sausage. How many people preferred sausage?

Answers

To find out how many people preferred sausage, you can start by multiplying the percentage who chose sausage, which is 30%, by the total number of people surveyed, which is 40.

So, 30% of 40 is:

0.30 x 40 = 12

Therefore, 12 people preferred sausage.

Answer: 12 people

Step-by-step explanation:

0.30 x 40 = 12

A country recently had a GDP of $1000 billion. Its consumption expenditures were $650 billion, its government spent $250 billion, and it had domestic investment of $150 billion. What was the value of this country’s net capital outflow?

Answers

The value of this country's net capital outflow is $200 billion.

We have,

The formula for net capital outflow (NCO) is:

NCO = Domestic Investment - Foreign Investment

Since the problem only gives us information about domestic investment, we need to use another formula to calculate foreign investment.

The formula for national saving (S) is:

S = GDP - Consumption Expenditures - Government Spending

We can rearrange this formula to solve for foreign investment:

Foreign Investment = GDP - Consumption Expenditures - Government Spending - Domestic Investment

Substituting the given values, we get:

Foreign Investment = $1000 billion - $650 billion - $250 billion - $150 billion

Foreign Investment = $1000 billion - $1050 billion

Foreign Investment = -$50 billion

The negative sign indicates that there is a net capital inflow (i.e., foreign investment is greater than domestic investment).

Therefore, the value of this country's net capital outflow is:

NCO = Domestic Investment - Foreign Investment

NCO = $150 billion - (-$50 billion)

NCO = $150 billion + $50 billion

NCO = $200 billion

Thus,

The value of this country's net capital outflow is $200 billion.

Learn more about net capital here:

https://brainly.com/question/18720725

#SPJ1

math hw for tonight
help solve this problem! Thank you!
ap cal bc

Answers

Answer:

first option

Step-by-step explanation:

differentiate using the power rule

[tex]\frac{d}{dx}[/tex] (a[tex]x^{n}[/tex] ) = na[tex]x^{n-1}[/tex]

then

[tex]\frac{dy}{dx}[/tex] = [tex]\frac{dy}{dt}[/tex] × [tex]\frac{dt}{dx}[/tex] = [tex]\frac{\frac{dy}{dt} }{\frac{dx}{dt} }[/tex]

y = t² + 4t

[tex]\frac{dy}{dt}[/tex] = 2t + 4

x = t - 3

[tex]\frac{dx}{dt}[/tex] = 1

then

[tex]\frac{dy}{dx}[/tex] = [tex]\frac{2t+4}{1}[/tex] = 2t + 4

Answer:

2t + 4

Step-by-step explanation:

A parametric equation is one where x and y are defined separately in terms of a third variable (often the parameter t).

To find dy/dx from parametric equations, differentiate each equation with respect to the parameter t, then use the chain rule:

[tex]\boxed{\dfrac{\text{d}y}{\text{d}x}=\dfrac{\text{d}y}{\text{d}t} \times \dfrac{\text{d}t}{\text{d}x}}[/tex]

Differentiate the two parametric equations with respect to t:

[tex]x=t-3 \implies \dfrac{\text{d}x}{\text{d}t}=1[/tex]

[tex]y=t^2+4t \implies \dfrac{\text{d}y}{\text{d}t}=2t+4[/tex]

Use the chain rule to combine them:

[tex]\begin{aligned}\implies \dfrac{\text{d}y}{\text{d}x}&=\dfrac{\text{d}y}{\text{d}t} \times \dfrac{\text{d}t}{\text{d}x}\\\\&=(2t+4) \times \dfrac{1}{1}\\\\&=2t+4\end{aligned}[/tex]

Therefore:

[tex]\boxed{\dfrac{\text{d}y}{\text{d}x}=2t+4}[/tex]

Ayuda por favor es para mañana, fracciones equivalentes. Doy coronita

Answers

These fractions are equivalent fractions by algebraic property:

Case 1: YES

Case 2: NO

Case 3: YES

Case 4: NO

Case 5: YES

Case 6: YES

Case 7: NO

Case 8: YES

Case 9: YES

Case 10: YES

Case 11: NO

Case 12: YES

Case 13: NO

Case 14: YES

Case 15: YES

Case 16: YES

Case 17: NO

Case 18: YES

How to determine if two fractions are equivalent

In this question we must check 18 cases of equivalent fractions, two fractions are equivalent if the following algebraic property is met:

a / b = (a · c) / (b · c), where a, b, c are integers and c is nonzero.

Now we proceed to determine if each pair is equivalent:

Case 1

2 / 3 = (2 · 2) / (3 · 2)

2 / 3 = 4 / 6 (YES)

Case 2

2 / 6 = (2 · 3) / (6 · 3)

2 / 6 = 6 / 18 (NO)

Case 3

9 / 9 = (9 · 4) / (9 · 4) = 36 / 36 (YES)

Case 4

3 / 11 = (3 · 3) / (11 · 3) = 9 / 33 (NO)

Case 5

7 / 8 = (7 · 2) / (8 · 2) = 14 / 16 (YES)

Case 6

4 / 6 = (4 · 5) / (6 · 5) = 20 / 30 (YES)

Case 7

5 / 6 = (5 · 2) / (6 · 2) = 10 / 12 (NO)

Case 8

2 / 7 = (2 · 4) / (7 · 4) = 8 / 28 (YES)

Case 9

6 / 12 = (6 · 2) / (12 · 2) = 12 / 24 (YES)

Case 10

4 / 9 = (4 · 5) / (9 · 5) = 20 / 45 (YES)

Case 11

9 / 10 = (9 · 3) / (10 · 3) = 27 / 30 (NO)

Case 12

1 / 5 = (1 · 5) / (5 · 5) = 5 / 25 (YES)

Case 13

12 / 12 = (12 · 3) / (12 · 3) = 36 / 36 (NO)

Case 14

8 / 11 = (8 · 4) / (11 · 4) = 32 / 44 (YES)

Case 15

5 / 5 = (5 · 4) / (5 · 4) = 20 / 20 (YES)

Case 16

6 / 9 = (6 · 4) / (9 · 4) = 24 / 36 (YES)

Case 17

3 / 7 = (3 · 8) / (7 · 8) = 24 / 56 (NO)

Case 18

10 / 12 = (10 · 4) / (12 · 4) = 40 / 48 (YES)

To learn more on equivalent fractions: https://brainly.com/question/29775885

#SPJ1

Please help asap!!!!! I'm confused

Answers

The area of the parallelogram is 8.5 square miles. This is found by multiplying the length of one of the parallel sides, 2 miles, by the height, which is given as 4 1/4 miles.

To find the area of a parallelogram, we can multiply the length of one of its parallel sides by the length of its perpendicular height. Therefore, to find the area of this parallelogram, we need to determine its height.

We are given that one of the parallel sides has a length of 4 1/4 mi and the other has a length of 2 mi. We are also given that the length of the perpendicular on one of the parallel sides is 4 1/4 mi, which means that this is the height of the parallelogram.

So, the area of the parallelogram is

Area = base x height

Area = 2 mi x 4 1/4 mi

Area = 8 1/2 mi²

Therefore, the area of the parallelogram is 8 1/2 square miles or 8.5 square miles.

To know more about Area of parallelogram:

https://brainly.com/question/19187448

#SPJ1

Can I get some help in this question?

What value of x satisfies the equation (7q^3x)^3=343q^36

Answers

4 is the value of the variable x.

What is algebraic Expression?

Any mathematical statement that includes numbers, variables, and an arithmetic operation between them is known as an expression or algebraic expression. In the phrase 4m + 5, for instance, the terms 4m and 5 are separated from the variable m by the arithmetic sign +.

We can simplify the left side of the equation by using the properties of exponents:

[tex](7q^{3x})^3 = 7^3 * (q^{3x})^3 = 343q^{9x}\\\\343q^{9x} = 343q^{36}\\\\q^{9x} = q^{36[/tex]

Now we can use the property that if [tex]a^b = a^c[/tex], then b = c. Therefore:

9x = 36

Dividing both sides by 9, we get:

x = 4

Therefore, the value of x that satisfies the equation is 4.

Learn more about algebraic Expression here:

https://brainly.com/question/953809

#SPJ1

The radius, R, of a sphere is 9.5cm . Calculate the sphere's volume,V . Use the value 3.14 for , and round your answer to the nearest tenth. (Do not round any intermediate computations.)

Answers

The volume of a sphere of radius 9.5 cm is approximately 3589.5 cm³

Calculating the volume of a sphere

From the question, we are to calculate the volume of a sphere

The volume of a sphere is given by the formula

V = 4/3 πr³

Where V is the volume of the sphere

and r is the radius of the sphere

From the given information,

r = 9.5 cm

Thus,

Volume of the sphere = 4/3 × π × 9.5³

Put π = 3.14

Volume of the sphere = 4/3 × 3.14 × 9.5³

Volume of the sphere = 3589.54333 cm³

Volume of the sphere ≈ 3589.5 cm³

Hence,

The volume of the sphere is 3589.5 cm³

Learn more on Calculating volume of a sphere here: https://brainly.com/question/10171109

#SPJ1

In parallelogram ABCD, AE =3x +7and CE=x+25.
What is AC?
O 68
O 34
O 18
09
D
C
B

Answers

In parallelogram ABCD, AE = CE since opposite sides of a parallelogram are congruent. Therefore, 3x + 7 = x + 25. Solving for x, we get x = 9. Substituting x = 9 into either AE or CE gives us the length of AC. So, AC = 3x + 7 = 3(9) + 7 = 34. Therefore, the answer is B. 34.

Please help solve for x image attached!

Answers

Answer:

7 should go in the green blank.

10/x = x/7

Please help on this asap!! 65 Points

Answers

No, It is not possible for 75% of the people surveyed at the park to purchase any two combination of the treats unless 14 people from the 'No preference group" decide to pick a treat. The two best frozen treat to pick will be Ice-cream sandwich and frozen fruit bar. With these two, 174 people are guaranteed to purchase the product.

How do we find the 75% chance of people surveyed buying any two combination of frozen treats?

To find the 75% chance of people surveyed buying any two combination we say

58 + 95 + 79 + 17 = 250

75% of 250 = 187.5 which is 188

We need to see if 188 of people will pick up a treat based on the survey

58 + 95 = 153 < 188

95 + 79 = 174 < 188  however, if  14 people from no preference group purchase, then, this is the best combination.

58 + 79 =  137 < 188

The answer provided is based on the question below as seen in the picture;

Is it possible to select a combination of two frozen treats so that 75% of the people surveyed would be able to purchase their favorite? If so, which two types of frozen treats should you select? Use words and numbers to justify your answer

Find more exercises on survey population;

https://brainly.com/question/1398803

#SPJ1

A group consists of seven Democrats and eight Republicans. Four people are selected to attend a conference.
a. In how many ways can four people be selected from this group of fifteen?
b. In how many ways can four Republicans be selected from the eight Republicans?
c. Find the probability that the selected group will consist of all Republicans.
a. The number of ways to select four people from the group of fifteen is
b. The number of ways to select four Republicans from the group of eight Republicans is
c. The probability is

Answers

There 1365 ways to choose four people from the group of fifteen.

b. There are 70 ways to choose  four Republicans from the group of eight Republicans.

C. The probability is about  0.0513, or  5.13%.

What is the probability  about?

a.  To know the ways that four people can be selected from this group of fifteen is by:

nCr = n! / (r! x (n-r)!),

Where:

n  = total number of items

r = is the number of items to be selected,

!  = the factorial of a number.

Putting in the values into the the formula:

15C4 = 15! / (4! x (15-4)!)

(15-4)! = 11!

15C4 = 1365

B.  Since:

n = 8

r = 4

Putting in the values into the the formula:

8C4 = 8! / (4!  x (8-4)!)

(8-4)! = 4!

8C4 = 70

c. The  Probability = Number of ways to choose four Republicans / Number of ways to choose four people

Hence  Probability = 70 / 1365

                             =  0.0513

Therefore, the probability that the selected group will consist of all Republicans is about  0.0513, or  5.13%.

Learn more about probability  from

https://brainly.com/question/24756209

#SPJ1

A virus takes 5 days to grow from 180 to 230. How many days will it take to grow from 180 to 260? Round to the nearest whole number.

Answers

It will take approximately 8 days for the virus to grow from 180 to 260.

We can set up a proportion to solve this problem. Let "x" represent the number of days it will take for the virus to grow from 180 to 260.

The proportion can be set up as follows;

(Change in value) / (Time taken) = (Change in value) / (Time taken)

Using the given information, we have;

(260 - 180) / x = (230 - 180) / 5

Simplifying the fractions on both sides of the equation, we get:

80 / x = 50 / 5

Cross-multiplying, we have;

80 × 5 = 50 × x

400 = 50x

Dividing both sides of the equation by 50, we get;

x = 400 / 50

x = 8

Therefore, it will take 8 days.

To know more about virus here

https://brainly.com/question/28067273

#SPJ1

A curve, described by x2 + y2 + 6y = 0, has a point A at (−3, −3) on the curve.

Part A: What are the polar coordinates of A? Give an exact answer.

Part B: What is the polar form of the equation? What type of polar curve is this?

Part C: What is the directed distance when theta equals 4 pi over 3 question mark Give an exact answer.

Answers

a) The polar coordinates of point A are (√(18), π/4).

b) The curve is a circle centered at the origin with radius 6.

c) The directed distance is the value of r, which is 6 √(3).

To find the polar coordinates of point A on the curve, we need to convert the point from Cartesian to polar coordinates. The conversion formula is:

r = √(x² + y²)

θ = arctan(y/x)

Using the values of point A, we have:

r = √((-3)² + (-3)²) = √(18)

θ = arctan((-3)/(-3)) = arctan(1) = π/4

To find the polar form of the equation x² + y² + 6y = 0, we need to convert it from Cartesian to polar coordinates. The conversion formulae are:

x = r cos(θ)

y = r sin(θ)

Using these formulae, we can rewrite the equation as:

r² cos²(θ) + r² sin²(θ) + 6r sin(θ) = 0

Simplifying this equation, we get:

r = -6 sin(θ) / (1 - cos²(θ))

To find the directed distance when θ equals 4 π over 3, we need to substitute this value of θ into the polar equation we found in Part B. Doing so, we get:

r = -6 sin(4 π/3) / (1 - cos²(4 π/3))

r = -6(-√(3)/2) / (1 - (-1/2)²)

r = 6 √(3)

To know more about polar coordinates here

https://brainly.com/question/31422978

#SPJ1

What is the value of z?

Answers

Answer: z = 8

Step-by-step explanation:

The diagram shows us that 8z + 3z + 2 = 90

so we can say that: 11z = 88

therefore z = 8.

Note: diagrams can be misleading! this diagram technically shows us that 64 < 26!

full method please

-6+28÷(-4)​

Answers

Answer:

-13

Step-by-step explanation:

To add fractions, find the lowest common denominator and then combine

The following blueprint of a kitchen has dimensions of 7 inches by 7 inches. The island has been highlighted in red.


The island's actual dimensions are 3 1/2 feet by 1 3/4 feet. If the scale of the blueprint is 1 inch = 2 feet, what are the dimensions of the island on the blueprint?

Answers

The dimensions of the island on the blueprint are 14 inches by 3.5 inches.

We have,

The actual dimensions of the island are 3 1/2 feet by 1 3/4 feet.

We need to find the dimensions of the island on the blueprint, given that the scale of the blueprint is 1 inch = 2 feet.

To convert the actual dimensions to the dimensions on the blueprint, we need to use the scale factor of 1 inch = 2 feet.

We can set up a proportion to relate the actual dimensions to the dimensions on the blueprint:

Actual dimension/blueprint dimension = scale factor

Let x be the length of the island on the blueprint.

Then we can set up the following proportion:

3.5 feet / (1.75 feet)

= x inches / 7 inches

Simplifying,

2 = x / 7

Multiplying both sides by 7, we get:

x = 14 inches

The length of the island on the blueprint is 14 inches.

Similarly, we can find the width of the island on the blueprint:

1.75 feet / 3.5 feet

= y inches / 7 inches

Simplifying, we have:

0.5 = y / 7

Multiplying both sides by 7, we get:

y = 3.5 inches

The width of the island on the blueprint is 3.5 inches.

Thus,

The dimensions of the island on the blueprint are 14 inches by 3.5 inches.

Learn more about expressions here:

https://brainly.com/question/3118662

#SPJ1

An insurance company offers an ordinary annuity that earns 6.5% interest compounded annually. A couple plans to make equal annual deposits into this account for 30 years and then make 20 equal annual withdrawals of €25,000, reducing the balance of the account to zero.

(i) Compute the value of the fund based on the withdrawals required. [5 marks]

(ii) Compute the amount of each deposit needed in order to maintain the fund. [5 marks]

(iii) Compute the total interest earned over the entire 50 years. [5 marks]​

Answers

Answer:

(i) To compute the value of the fund based on the withdrawals required, we can use the formula for the future value of an annuity due:

FV = P * ((1 + r)^n - 1) / r) * (1 + r)

where FV is the future value of the annuity, P is the annual payment, r is the interest rate per period, n is the total number of periods, and the extra (1 + r) factor is because the payments are made at the beginning of each period.

In this case, P = €25,000, r = 0.065, n = 20. We want to find the future value at the end of the 20-year period:

FV = 25000 * ((1 + 0.065)^20 - 1) / 0.065) * (1 + 0.065)

FV ≈ €743,704.96

Therefore, the value of the fund based on the withdrawals required is approximately €743,704.96.

(ii) To compute the amount of each deposit needed in order to maintain the fund, we can use the formula for the present value of an ordinary annuity:

PV = P * ((1 - (1 + r)^(-n)) / r)

where PV is the present value of the annuity, P is the annual payment, r is the interest rate per period, and n is the total number of periods.

In this case, PV = €743,704.96, r = 0.065, n = 20. We want to find the annual payment:

PV = P * ((1 - (1 + 0.065)^(-20)) / 0.065)

P ≈ €22,630.53

Therefore, the amount of each deposit needed in order to maintain the fund is approximately €22,630.53.

(iii) To compute the total interest earned over the entire 50 years, we can subtract the total deposits from the total withdrawals, and then subtract the initial balance. The total deposits are the annual deposit amount times the number of years (30), and the total withdrawals are the annual withdrawal amount times the number of years (20). The initial balance is the present value of the annuity that we calculated in part (ii).

Total deposits = €22,630.53 * 30 = €678,915.90

Total withdrawals = €25,000 * 20 = €500,000

Initial balance = €743,704.96

Total interest earned = Total withdrawals - Total deposits - Initial balance

Total interest earned = €500,000 - €678,915.90 - €743,704.96

Total interest earned ≈ -€922,620.86

Note that the negative sign indicates that the insurance company actually earned interest on this annuity, rather than the couple earning interest on their investment. This is because the withdrawals are greater than the deposits, and the interest rate earned by the insurance company is greater than the interest rate paid to the couple.

Step-by-step explanation:

i need help with this prblem

Answers

Answer:

y = x + 2

Step-by-step explanation:

y = mx + b

Point (0, 2) shows that the y-intercept is 2, so b = 2.

y = mx + 2

Now we find the slope using the 2 points, (0, 2) and (2, 4).

m = (4 - 2)/(2 - 0) = 1

y = x + 2

I need help as soon as possible please??

Answers

Answer:

4x² - 11xy - 3y²

Step-by-step explanation:

(4x + y)(x - 3y) =

Multiply each term of the first binomial by each term of the second binomial.

= 4x² - 12xy + xy - 3y²

Now combine like terms.

= 4x² - 11xy - 3y²

Try it
Mrs. Chauvet has an unfair number cube that lands with 6 For how many of the outcomes does X = 0, meaning
facing up 40% of the time.
the outcome has no 6s?
Let X = the number of times that she rolls a 6 among 3
trials.
For how many of the outcomes does X = 1, meaning
the outcome has exactly one 6?
For how many of the outcomes does X=2, meaning
the outcome has exactly two 6s?
For how many of the outcomes does X=3, meaning
the outcome has exactly three 6s?

Answers

Answer:

The problem describes rolling an unfair number cube which lands with 6, and asks for the number of outcomes where X=0, X=1, X=2, and X=3.

X can be 0, 1, 2, or 3 if X equals the number of times she rolls a 6 over the course of three tries.

We must count the instances where none of the three trials yields a six in order to determine the number of occurrences where X = 0. Since there is a 0.4 percent chance that the cube will fall on 6, there is a 0.6 percent chance that it won't. Therefore, there are 0.6 * 3 = 0.216, or 21.6%, of outcomes where X = 0.

To find the number of outcomes where X=1, we need to count the number of outcomes where exactly one of the three trials results in a 6. There are three ways to choose which trial will result in a 6, and each of the other two trials must not result in a 6. Therefore, the number of outcomes where X=1 is 3 × 0.4 × 0.6^2 = 0.432 or 43.2%.

We must count the outcomes where precisely two out of the three trials yield a six in order to determine the number of events where X=2. There are three options for selecting the two trials that will end in a 6, and the third trial cannot also end in a 6. The proportion of outcomes where X=2 is therefore 3 0.4 2 0.6 = 0.288 or 28.8%.

Finally, to find the number of outcomes where X=3, we need to count the number of outcomes where all three trials result in a 6. This occurs with probability 0.4^3 = 0.064 or 6.4%.

Therefore, the number of outcomes where X = 0 is 21.6%, X=1 is 43.2%, X=2 is 28.8%, and X=3 is 6.4%.

The distance between cities A and B on a map is 12.5 in. The distance from city B to city C, is 8.5 in, and the distance from C to A is 16.25 in. If the bearing
from A to B is N75°E, find the bearing from C to 4. Round to the nearest tenth of a degree.
16.25 in
12.5 in
8.5 in
The bearing from city C to city 4 is approximately (Choose one) (Choose one)

Answers

The bearing of C to A is 239⁰.

What is the bearing of C to A?

The bearing of C to A is calculated by finding the value of angle C using cosine rule since we know the value of all the sides of the triangle.

AB² = AC² + CB² - 2(AC)(CB) cosC

12.5² = 16.25² + 8.5² - 2(16.25 x 8.5) x cos C

156.25 = 336.3125 - 276.25cosC

276.25cosC = 180.06

cosC = 180.06/276.25

cos (C) = 0.6518

C = cos⁻(0.6518)

C = 49.3⁰

The value of angle A is calculated as follows;

Sin A/CB = Sin C/AB

sin A/8.5 = sin 49.3/12.5

sin A = 8.5 [sin 49.3/12.5]

sin A = 0.5157

A = sin⁻¹ (0.5157)

A = 31⁰

The bearing of C to A is calculated as;

= 270⁰ - 31⁰

= 239⁰

Learn more about bearing here: https://brainly.com/question/28782815

#SPJ1

(Identifying Transformations LC)

Use the image to determine the type of transformation shown.

Preimage of polygon ABCD. A second image, polygon A prime B prime C prime D prime to the right of the first image with all points in the same position.

Horizontal translation
Vertical translation
Reflection across the x-axis
90° clockwise rotation

Answers

If the polygon A, B, C, D is transformed to polygon A', B', C', D' to the right of the first image with all points in the same position, then the transformation is a horizontal translation.

Option A is the correct answer.

We have,

A horizontal translation moves every point of a figure the same distance in the same direction.

In this case, since the second polygon is to the right of the first one, we know that every point of the polygon has been translated to the right by the same amount.

A vertical translation would move every point of the figure the same distance in the same direction, but vertically instead of horizontally.

A reflection across the x-axis would flip the figure over the x-axis, so points that were above the x-axis would end up below it and vice versa.

A 90° clockwise rotation would rotate the figure 90 degrees to the right.

Thus,

If the polygon A, B, C, D is transformed to polygon A', B', C', D' to the right of the first image with all points in the same position, then the transformation is a horizontal translation.

Learn more about translation here:

https://brainly.com/question/12463306

#SPJ1

Question 1 (1 point)
Write an inequality for the sentence.
The stadium held less than 25,000.
B
O a
O b
9ba580e611107c96c9efb866417dc160.webm 64 KB
s> 25,000
$≤25,000
Oc $<25,000

Answers

The inequality that represents the sentence "The stadium held less than 25,000 people" is given as follows:

c < 25,000.

What are the inequality symbols?

The four inequality symbols, along with their meaning on the number line and the coordinate plane, are presented as follows:

> x: the amount is greater than x -> the number is to the right of x with an open dot at the number line. -> points above the dashed horizontal line y = x on the coordinate plane.< x: the amount is less than x. -> the number is to the left of x with an open dot at the number line. -> points below the dashed horizontal line y = x on the coordinate plane.≥ x: the amount is at least x. -> the number is to the right of x with a closed dot at the number line. -> points above the solid vertical line y = x on the coordinate plane.≤ the amount is at most x. -> the number is to the left of x with a closed dot at the number line. -> points above the dashed vertical line y = x on the coordinate plane.

The amount is less than in this problem, hence the symbol is given as follows:

<.

As the amount is less than 25000, the inequality is given as follows:

c < 25,000.

More can be learned about inequalities at brainly.com/question/25275758

#SPJ1

Other Questions
A focus group usually involves how many subjects A 1 or 2 B 3 to 6 C 7 to 10 True or False A variable associates a name with a value, making it easy to remember and use the value later in aprogram. According to the text, the cognitive-behavioral approaches to treating schizotypal personality disorder are similar to cognitive-behavioral approaches to treatingA) psychotic disorders.B) obsessive compulsive disorder.C) depression. D) autism. a long rectangular channel that is 8 m wide and has a mild slope ends in a free outfall. if the water depth at the brink is 0.55 m, what is the discharge in the channel? Analyzing transactions is theA. secondB. thirdC. firstD. laststep of the accounting cycle. Which equation represents the slope-intercept form of the line below Ethylene is the smallest member in the family of _____ containing a carbon-carbon _____ bond. Need help asap please!!Answer the questions below:What are three ways an author can portray imagery via the written word?List one example from Act II where Shakespeare uses imagery.List one example from Act II that highlights a motif of the play.(A MIDSUMMER NIGHT'S DREAM, ACT II: MOTIF AND IMAGERY) a researcher wishes to determine whether people with high blood pressure can reduce their blood pressure by following a particular diet. subjects were randomly assigned to either a treatment group or a control group. the mean blood pressure was determined for each group, and a 95% confidence interval for the difference in the means for the treatment group versus the control group, , was found to be . give an interpretation of this confidence interval. Which organisms obtain its energy mostly from dead or decaying matter (which kingdom)? The shaft is made from a solid steel section AB and a tubular portion made of steel and having a brass core. If it is fixed to a rigid support at A, and a torque of T = 50 lb. Ft is applied to it at C, determine the angle of twist that occurs at C and compute the maximum shear stress and maximum shear strain in the brass and steel. Given Gst = 11,500 ksi and Gbr = 5600 ksi help please! we have to look at the pyramid and create a claim about how we think energy will change and if there is the same amount of energy at each level This is an trick question what is there but you can't see it Is this a action or regulation?1. Stores may not sell alcoholic beverages to people under age 212. A factory that pollutes a stream must pay to clean up the stream3. A country decides to plant 100 trees to improve the city park4. The department of education pays for rural schools to install solar panels5. A school district provides students transportation to and from school 6. Children under age 16 must attend school7. A state pays for children to have free vaccinations 8. A county offers employment and money management counseling to low income families9. Farmers may not use certain dangerous pesticides on their crops 10. A state adoption agent must not consider race when placing a child for adoption What are the characteristics of Cutaneous Leishmaniasis? Handel and J. S. Bach met often and collaborated on works.T or F What information do you need before you can decide which type of business might be the most successful? What is meant by "scope of practice" for a health care professional? what are 5 physical manifestations and 3 serious possibilities of inhalant intoxication? (NANDD DDP) If a new cash deposit creates excess reserves of$5,000$5,000and the required reserve ratio is1010percent, the banking system can increase the money supply by a maximum of:a.$50,000$50,000.b.$500$500.c.$5,000$5,000.d.$4,500$4,500.